Đến nội dung

Nhok Tung

Nhok Tung

Đăng ký: 25-04-2015
Offline Đăng nhập: 27-06-2021 - 20:46
***--

#647159 $CMR : \frac{1}{\sqrt{1+x^2}} +...

Gửi bởi Nhok Tung trong 30-07-2016 - 10:14

Dấu $\leq$ thì BĐT đúng, còn $\geq$ thì với x =1 y =2 BĐT sai  :D




#635619 $(a+b+c)^{5}\geq 81abc(a^{2}+b^{2}+c^...

Gửi bởi Nhok Tung trong 25-05-2016 - 23:18

Tiếp theo là 1 bài tương tự như bài này :

CMR với a,b,c>0 ,abc=1,ta có:

 

$\frac{a+b+c}{3}\geq \sqrt[5]{\frac{a^{2}+b^{2}+c^{2}}{3}}$

Thực chất 2 bài là 1 thôi  :D




#635618 $(a+b+c)^{5}\geq 81abc(a^{2}+b^{2}+c^...

Gửi bởi Nhok Tung trong 25-05-2016 - 23:16

CMR với mọi a,b,c dương ta có:

$(a+b+c)^{5}\geq 81abc(a^{2}+b^{2}+c^{2})$

Ta có bđt phụ : $3abc\leq \frac{(ab+bc+ca)^{2}}{a+b+c}$

Do đó VP $\leq$  $\frac{27(ab+bc+ca)^{2}}{a+b+c}(a^{2}+b^{2}+c^{2}) =\frac{27}{a+b+c}.(ab+bc+ca)(ab+bc+ca)(a^{2}+b^{2}+c^{2})\leq      \frac{27}{a+b+c}.\frac{[a^{2}+b^{}+c^{2}+2(ab+bc+ca)]^{3}}{27} =\frac{27}{a+b+c}.\frac{(a+b+c)^{6}}{27}=(a+b+c)^{5}$

=> đpcm




#635587 $\sqrt{\frac{a+b}{c+ab}}+\s...

Gửi bởi Nhok Tung trong 25-05-2016 - 22:23

Bài 1: Cho a,b,c > 0 và ab+bc+ac > 0

Chứng minh: $\sqrt{\frac{1+a^{2}}{b+c}}+\sqrt{\frac{1+b^{2}}{a+c}}+\sqrt{\frac{1+c^{2}}{a+b}}\geq 3$

$\sum \sqrt{\frac{1+a^{2}}{b+c}}\geq 3\sqrt[6]{\prod \frac{1+a^{2}}{b+c}}$

cần chứng minh $\prod (1+a^{2})\geq \prod (a+b)$

Ta có : $(1+a^{2})(1+b^{2})\geq (a+b)^{2}$

thiết lập tương tự rồi nhân vế theo vế các bđt, ta có đpcm

Ở đây a,b,c >0 thì ab+bc+ca >0 rồi  :D  :D  :D




#635581 $\sqrt{\frac{a+b}{c+ab}}+\s...

Gửi bởi Nhok Tung trong 25-05-2016 - 22:17

Bài 2: cho a+b+c = 3

chứng minh  $\sqrt{\frac{a+b}{c+ab}}+\sqrt{\frac{b+c}{a+bc}}+\sqrt{\frac{a+c}{b+ac}}\geq 3$

Áp dụng BĐT AM-GM :

$\sum \sqrt{\frac{a+b}{c+ab}}\geq 3\sqrt[6]{\prod \frac{a+b}{c+ab}}$

Cần chứng minh $(a+b)(b+c)(c+a)\geq (c+ab)(b+ac)(a+bc)$

Ta có :

$(c+ab)(a+bc)(b+ca)\leq \frac{1}{8}(a+b)(b+c)(c+a)(1+a)(1+b)(1+c) \leq \frac{1}{8}(a+b)(b+c)(c+a)\frac{1}{27}(1+1+1+a+b+c)^{3}=(a+b)(b+c)(c+a)$

-> đpcm




#631970 $\frac{x^{m}}{y^{n}}+\...

Gửi bởi Nhok Tung trong 08-05-2016 - 17:26

Cho x,y,z > 0, xyz=1. Chứng minh :$\frac{x^{m}}{y^{n}}+\frac{y^{m}}{z^{n}}+\frac{z^{m}}{x^{n}}\geq x+y+z$




#630554 P=$\sqrt{\frac{a^{3}}{a^{3}+(b+c)^{3}}}+\sqrt{\frac{...

Gửi bởi Nhok Tung trong 01-05-2016 - 16:53

cho a,b,c >0. cmr

P=$\sqrt{\frac{a^{3}}{a^{3}+(b+c)^{3}}}+\sqrt{\frac{b^{3}}{b^{3}+(a+c)^{3}}}+\sqrt{\frac{c^{3}}{c^{3}+(b+a)^{3}}}\geq 1$

$\sqrt{\frac{a^{3}}{a^{3}+(b+c)^{3}}}=\frac{1}{\sqrt{1+\left ( \frac{b+c}{a} \right )^{3}}}=\frac{1}{\sqrt{(1+\frac{b+c}{a})[(\frac{b+c}{a})^{2}-\frac{b+c}{a}+1]}}\geq \frac{2}{(\frac{b+c}{a})^{2}+2}=\frac{2a^{2}}{(b+c)^{2}+2a^{2}}\geq \frac{2a^{2}}{2(b^{2}+c^{2})+2a^{2}}=\frac{a^{2}}{a^{2}+b^{2}+c^{2}}$

Tương tự : $\sqrt{\frac{b^{3}}{b^{3}+(c+a)^{3}}}\geq \frac{b^{2}}{a^{2}+b^{2}+c^{2}}$

$\sqrt{\frac{c^{3}}{c^{3}+(a+b)^{3}}}\geq \frac{c^{2}}{a^{2}+b^{2}+c^{2}}$

Cộng vế theo vế các BĐT trên ta có đpcm

Đẳng thức xảy ra khi a=b=c




#630474 Tìm GTNN của $P=\frac{bc}{a}+\frac{ca...

Gửi bởi Nhok Tung trong 01-05-2016 - 07:18

Cho a, b, c > 0 thỏa mãn $a^{2}+b^{2}+c^{2}=1$. Tìm GTNN của $P=\frac{bc}{a}+\frac{ca}{b}+\frac{ab}{c}$

đặt $\frac{ab}{c}=x,\frac{bc}{a}=y,\frac{ca}{b}=z$

Ta có xy+yz+xz=1

P = x + y + z $\geq \sqrt{3(ab+bc+ca)}$=$\sqrt{3}$




#630429 Tính : $ L=\lim_{x\rightarrow 0}\frac{...

Gửi bởi Nhok Tung trong 30-04-2016 - 21:57

Tính : $ L=\lim_{x\rightarrow 0}\frac{\sqrt{1+2x}-\sqrt[3]{1+3x}}{x} $

$L=\lim_{x\rightarrow 0}\left ( \frac{\sqrt{1+2x}-1}{x}-\frac{\sqrt[3]{1+3x}-1}{x} \right ) =\lim_{x\rightarrow 0}\left [ \frac{2}{\sqrt{1+2x}+1}-\frac{3}{\sqrt[3]{(1+3x)^{2}}+\sqrt[3]{1+3x}+1} \right ]=1-1=0$




#629685 $\frac{1}{a+b+4}+\frac{1}{b...

Gửi bởi Nhok Tung trong 26-04-2016 - 17:06

Cho a,b,c là các số thực dương thõa mãn$abc=1$. Chứng minh rằng:

$\frac{1}{a+b+4}+\frac{1}{b+c+4}+\frac{1}{c+a+4}\le \frac{1}{2}$

Ta có $\frac{1}{a+b+4}\leq \frac{1}{4}\left ( \frac{1}{a+2}+\frac{1}{b+2} \right )$

Do đó VT $\leq \frac{1}{2}\sum \frac{1}{a+2}$

Ta chứng minh $\sum \frac{1}{a+2}\leq 1$

Biến đổi tương đương ta được $ab+bc+ca\geq 3$ ( đúng theo AM-GM)

Vậy BĐT được chứng minh

Đẳng thức xảy ra khi a=b=c=1




#629683 Có bao nhiêu tam giác tạo thành từ 8 đỉnh của đa giác 8 cạnh đều mà không có...

Gửi bởi Nhok Tung trong 26-04-2016 - 16:52

Có bao nhiêu tam giác tạo thành từ 8 đỉnh của đa giác 8 cạnh đều mà không có cạnh nào là cạnh của bát giác ấy.

Tổng quát của bài này :

Cho đa giác đều n cạnh, số đa giác có k cạnh ( k < n) nhận đỉnh là đỉnh của đa giác n cạnh sao cho không có cạnh nào là cạnh của đa giác này

$\frac{n}{k}C_{n-k-1}^{k-1}$




#629537 $\sum \frac{1}{(a+b)^{2}}\g...

Gửi bởi Nhok Tung trong 25-04-2016 - 19:05

Cho a,b,c > 0. Chứng minh rằng :

$\sum \frac{1}{(a+b)^{2}}\geq \frac{3\sqrt{3abc(a+b+c)}(a+b+c)^{2}}{4(ab+bc+ca)^{3}}$




#627825 $\frac{a}{b^{3}+ab}+\frac{b...

Gửi bởi Nhok Tung trong 17-04-2016 - 20:53

$\sum \frac{a}{b^{3}+ab}=\frac{1}{a}+\frac{1}{b}+\frac{1}{c}-\sum \frac{b}{a+b^{2}}\geq \sum \frac{1}{a}-\frac{1}{2}\sum \frac{1}{\sqrt{a}}\geq \frac{(\sum \frac{1}{\sqrt{a}})^{2}}{3}-\frac{1}{2}\sum \frac{1}{\sqrt{a}}$

Đặt $t=\sum \frac{1}{^{\sqrt{a}}}\geq 3$

Ta chứng minh $\frac{t^{2}}{3}-\frac{t}{2}\geq \frac{3}{2}\Leftrightarrow (t-3)(2t+3)\geq 0$ (TRUE)

BĐT đc chứng minh




#627796 CMR: $\sum \frac{a^3}{b^2+3}\geq \frac{3}{4}$

Gửi bởi Nhok Tung trong 17-04-2016 - 19:54

Cho $3$ số thực dương $a,b,c$ thỏa mãn $ab+bc+ca=3$. CMR:

$$\frac{a^3}{b^2+3}+\frac{b^3}{c^2+3}+\frac{c^3}{a^+3}\geq \frac{3}{2}$$                                

$\sum \frac{a^{3}}{b^{2}+3}=\sum \frac{a^{3}}{b^{2}+ab+bc+ca}=\sum \frac{a^{3}}{(a+b)(b+c)}$

Áp dụng BĐT AM-GM :

$\frac{a^{3}}{(b+a)(b+c)}+\frac{b+c}{8}+\frac{b+a}{8}\geq \frac{3a}{4}$

Tương tự. Cộng vế theo vế các BĐT ta được :

$\sum \frac{a^{3}}{(a+b)(b+c)}\geq \frac{a+b+c}{4}\geq \frac{\sqrt{3(ab+bc+ca)}}{4}=\frac{3}{4}$

Đẳng thức xảy ra <=> a=b=c=1

Bạn này gõ tiêu đề là 3/4 mà đề lại ghi 3/2  :D




#627777 $\sum \frac{1}{1+ab}\geq \frac...

Gửi bởi Nhok Tung trong 17-04-2016 - 18:03

1. Cho $a,b,c\geq 0,a+b+c=3$. Chứng minh :

$ab^{2}+bc^{2}+ca^{2}+abc\leq 4$

2. Cho a,b,c > 0, a+b+c = 3. Chứng minh :

$\sum \frac{1}{1+ab}\geq \frac{9}{2(\sqrt{a}+\sqrt{b}+\sqrt{c})}$